11th Class Physics Systems Of Particles & Rotational Motion / कणों के निकाय तथा घूर्णी गति Question Bank 11th CBSE Physics Rotational Motion

  • question_answer
    If angular momentum is conserved in a system whose moment of inertia is decreased, will its rotational kinetic energy be also conserved?  

    Answer:

                    Here, \[\text{L}=l\omega \]= constant \[\therefore \]  K.E. of rotation,                \[K=\frac{1}{2}I{{\omega }^{2}}\]                                                                 \[K=\frac{1}{2I}{{I}^{2}}{{\omega }^{2}}=\frac{{{L}^{2}}}{2I}\] As L is constant,                                                \[\therefore \]  \[K\propto 1/I\] When moment of inertia (I) decreases, K.E of rotation (K) increases. Thus K.E. of rotation is not conserved.


You need to login to perform this action.
You will be redirected in 3 sec spinner